1. #1
    hhsilver
    hhsilver's Avatar SBR PRO
    Join Date: 06-07-07
    Posts: 7,280
    Betpoints: 29922

    math question

    More than a month ago someone posed this challenge in players talk :

    (see this thread)>>http://www.sportsbookreview.com/forum/players-ta...-question.html

    Question in post 1. my answer on post 3.

    The "math guy" who started the thread never came back to it even though I bumped it a couple times.

    I really want to know if I'm correct. If not, I would like to know the answer and have it explained.

    I started another thread , http://www.sportsbookreview.com/forum/players-ta...athdotcom.html ,
    to try to get him to respond. He did with simply a "wrong" --- no correct answer, no explanation.

    I believe my answer may be correct and would like to show how I got it. If it's not correct , maybe someone here could show
    me why and give the solution.
    Thanks to any real math guys here who may care to respond.

  2. #2
    Kemalettin
    Kemalettin's Avatar Become A Pro!
    Join Date: 03-20-10
    Posts: 1,351
    Betpoints: 78

    he s stiff

  3. #3
    mathdotcom
    mathdotcom's Avatar Become A Pro!
    Join Date: 03-24-08
    Posts: 11,689
    Betpoints: 1943

    Quote Originally Posted by hhsilver View Post
    More than a month ago someone posed this challenge in players talk :

    (see this thread)>>http://www.sportsbookreview.com/forum/players-ta...-question.html

    Question in post 1. my answer on post 3.

    The "math guy" who started the thread never came back to it even though I bumped it a couple times.

    I really want to know if I'm correct. If not, I would like to know the answer and have it explained.

    I started another thread , http://www.sportsbookreview.com/forum/players-ta...athdotcom.html ,
    to try to get him to respond. He did with simply a "wrong" --- no correct answer, no explanation.

    I believe my answer may be correct and would like to show how I got it. If it's not correct , maybe someone here could show
    me why and give the solution.
    Thanks to any real math guys here who may care to respond.
    Then show it. No points for throwing a guess out there. Unreal you do not give up, you must be really fukkin barreled in.

  4. #4
    Kemalettin
    Kemalettin's Avatar Become A Pro!
    Join Date: 03-20-10
    Posts: 1,351
    Betpoints: 78

    hey freak post the answer then

  5. #5
    sharpcat
    sharpcat's Avatar Become A Pro!
    Join Date: 12-19-09
    Posts: 4,516

    I am not sure that I understand the problem trying to be solved

    But I would conclude that your answer is wrong because he is asking for the highest number and your solution is going to return a percentage.

  6. #6
    hhsilver
    hhsilver's Avatar SBR PRO
    Join Date: 06-07-07
    Posts: 7,280
    Betpoints: 29922

    ok - by show it, i guess you mean how i got it. I didn't think you were interested. About the barrelled in, I am not pressing this for the points, but for the answer. I haven't been able to find anything to verify my answer.

    here is why my answer is n/(n+1) ........

    for the uniform dist you described, I believe the expected result of n picks would have them be evenly spaced in the interval (in this case the interval is [0,1]

    1 pick >>>> exp pick is .5 , exp max is 1(1+1) = .5
    2 picks >>> exp results are .33333.. , .6666.... , exp max is 2/(2+1) = .66666
    ..... and so on..
    n picks >>> exp results are n evenly spaced points on the interval......, exp max is the largest of these, which is n/(n+1)

    I would appreciate your thoughts on this .... if it's wrong, please explain what the answer should be.

  7. #7
    hhsilver
    hhsilver's Avatar SBR PRO
    Join Date: 06-07-07
    Posts: 7,280
    Betpoints: 29922

    Quote Originally Posted by sharpcat View Post
    I am not sure that I understand the problem trying to be solved

    But I would conclude that your answer is wrong because he is asking for the highest number and your solution is going to return a percentage.
    not true ... my answer is a number, not a pct.

    Notice the interval is 0 to 1.

    n/(n+1) is a number in that interval

  8. #8
    Ganchrow
    Nolite te bastardes carborundorum.
    Ganchrow's Avatar Become A Pro!
    Join Date: 08-28-05
    Posts: 5,011
    Betpoints: 1088

    n n+1 is of course correct, just as hhsilver has quite cleverly intuited.

    No clue why mathdotcom gave him such a hard time.

    But just for shits and giggles, a rather informal late night mathematical proof might read something like this:

    We're looking for the E(sup(Xk)) where Xk is a vector of k realizations from the uniform distribution.

    For the degenerate case of k=1, the pdf(sup(X1)) = pdf(X1) = 1. Hence:

    = 1 2 * p2 | p=0 → 1
    = 1 2

    So E(sup(X1)) = E(X1) = 1 2

    For k=2:
    Pr(sup(X2) = p ∈ [0,1] )
    = Pr( (x(1) ≤ p AND x(2) = p) OR (x(1) = p AND x(2) ≤ p) ) (this is the key step)
    ≈ 2*p*Δ
    Hence:

    = 2 3 * p3 | p=0 → 1
    = 2 3
    Continuing with the same logic, it's clear that for k=n:

    Pr(sup(Xn) = p ∈ [0,1] )
    ≈ n*pn-1

    = n n+1 * pn+1 | p=0 → 1
    = n n+1

    QED

    Nicely done, hh. YOu get full credit in my book.

  9. #9
    aca
    aca's Avatar Become A Pro!
    Join Date: 03-20-06
    Posts: 2,111

    Thanks!

  10. #10
    Peeig
    Underbetting my roll
    Peeig's Avatar Become A Pro!
    Join Date: 02-06-08
    Posts: 567
    Betpoints: 1213

    Ganchrow is back???

  11. #11
    Patrick McIrish
    Patrick McIrish's Avatar Become A Pro!
    Join Date: 09-15-05
    Posts: 2,864
    Betpoints: 115

    Let's all hope so. John, apologize and tender an offer immediately! Thanks.



    Well done Silver, you get all the credit.

  12. #12
    mathdotcom
    mathdotcom's Avatar Become A Pro!
    Join Date: 03-24-08
    Posts: 11,689
    Betpoints: 1943

    Ganch I said in a previous thread the answer should have integrals in it.

    Others made intuitive guesses too, like 1 - (1/2)^n.

  13. #13
    hhsilver
    hhsilver's Avatar SBR PRO
    Join Date: 06-07-07
    Posts: 7,280
    Betpoints: 29922

    Welcome back, Ganch. Thank you for verifying my answer and for providing a proof.
    I am not sure "intuitive" is the right word. I guess it depends on the def of intuitive/intuition.
    One def is:
    intuition:
    instinctive knowing (without the use of rational processes)

    I don't think that applies in this case because I understand expectation and unform dist. I got my answer using rational processes based on my knowledge of those concepts. But I won't quibble with it being called intuition in the sense that my professors were fond of saying something was "intuitively obvious" when they wanted to skip a tedious proof of the "obvious".
    What I didn't like was mathguy equating "intuition" with a "wild guess". I realize I didn't offer a proof , but surely my reasoning indicates it wasn't a wild guess.

    Anyway, it's great to have you back. You were missed.

    I want to say a little more. I hope other will read this and follow the links to get the full story.

    Quote Originally Posted by mathdotcom View Post
    Ganch I said in a previous thread the answer should have integrals in it.

    Others made intuitive guesses too, like 1 - (1/2)^n.
    )

    This is simply not true. See the original question (unedited as of this writing) here:

    http://www.sportsbookreview.com/forum/players-ta...-question.html.

    No mention was made of using integration. A question was asked , the answer was given and ignored even after bumps and a pm.

    I started two other threads to try to get the answer:

    this thread and another one:

    http://www.sportsbookreview.com/forum/players-ta...athdotcom.html

    Notice, at one point, that mathguy said my answer was "wrong" . Then after I explained , at his request, how I got it, he said , still without saying the answer was correct, I had to use an integral. Here is his first mention of integrals - not in the original question as the quote above suggests.

    I liken this to the following:

    mathguy :: what is the area bounded by the x-axis, y=4, x=6 and x=11?

    responder:: (thinking - hey that's a rectangle) area is 4x5 = 20.

    mathguy :: wrong!!!! you didn't integrate .

    ------

  14. #14
    Ganchrow
    Nolite te bastardes carborundorum.
    Ganchrow's Avatar Become A Pro!
    Join Date: 08-28-05
    Posts: 5,011
    Betpoints: 1088

    Quote Originally Posted by hhsilver View Post
    Welcome back, Ganch. Thank you for verifying my answer and for providing a proof.
    I am not sure "intuitive" is the right word. I guess it depends on the def of intuitive/intuition.
    One def is:
    intuition:
    instinctive knowing (without the use of rational processes)
    intuition

    1. direct perception of truth, fact, etc., independent of any reasoning process; immediate apprehension.
    2. a fact, truth, etc., perceived in this way.
    3. a keen and quick insight.
    4. the quality or ability of having such direct perception or quick insight.


    I was using the verb intuit more in the sense of the 3rd and 4th definitions of the root noun.

Top